I Riemann curvature tensor derivation

cozycoz
Messages
13
Reaction score
1
Riemann tensor is defined mathematically like this:
##∇_k∇_jv_i-∇_j∇_kv_i={R^l}_{ijk}v_l##

Using covariant derivative formula for covariant tensors and covariant vectors. which are

##∇_av_b=∂_av_b-{Γ^c}_{ab}v_c##
##∇_aT_{bc}=∂_av_{bc}-{Γ^d}_{ac}v_{db}-{Γ^d}_{ab}v_{dc} ##,

I got these:

##∇_k∇_jv_i=∂_k∂_jv_i-∂_k{Γ^m}_{ji}v_m-{Γ^m}_{ji}∂_kv_m-{Γ^l}_{ki}∂_lv_j+{Γ^l}_{ki}{Γ^m}_{lj}v_m-{Γ^l}_{kj}∂_lv_i+{Γ^l}_{kj}{Γ^m}_{li}v_m##(I'll call each term by its number : ##∂_k∂_jv_i## is "1" because it's the first term)

##∇_j∇_kv_i=∂_j∂_kv_i-∂_j{Γ^m}_{ki}v_m-{Γ^m}_{ki}∂_jv_m-{Γ^l}_{ji}∂_lv_k+{Γ^l}_{ji}{Γ^m}_{lk}v_m-{Γ^l}_{jk}∂_lv_i+{Γ^l}_{jk}{Γ^m}_{li}v_m##

Then 1, 3, 4, 6, 7 terms all vanish when we substract the lower from the upper' according to my professor.

Especially he noted that 3rd term vanishes because it's basically the same when you just exchange the indices j⇔k and add up all m's.
But then isn't it also the case for the second term?In 2nd
##∂_j{Γ^m}_{ki}v_m##
##∂_k{Γ^m}_{ji}v_m## ,

And in 3rd
##{Γ^m}_{ki}∂_jv_m##
##{Γ^m}_{ji}∂_kv_m##
(They are the same)​

I see no reason why I can't apply the logic for 3rd to 2nd...could you help me please
 
Last edited:
Physics news on Phys.org
cozycoz said:
Riemann tensor is defined mathematically like this:
##∇_k∇_jv_i-∇_j∇_kv_i={R^l}_{ijk}v_l##

Using covariant derivative formula for covariant tensors and covariant vectors. which are

##∇_av_b=∂_av_b-{Γ^c}_{ab}v_c##
##∇_aT_{bc}=∂_av_{bc}-{Γ^d}_{ac}v_{db}-{Γ^d}_{ab}v_{dc} ##,
[..]
In 2nd
##∂_j{Γ^m}_{ki}v_m##
##∂_k{Γ^m}_{ji}v_m## ,

And in 3rd
##{Γ^m}_{ki}∂_jv_m##
##{Γ^m}_{ji}∂_kv_m##
(They are the same)​

I see no reason why I can't apply the logic for 3rd to 2nd...could you help me please

This ##∂_j{Γ^m}_{ki}v_m-∂_k{Γ^m}_{ji}v_m## can be written ##∂_{[j}{Γ^{|m|}}_{k]i}v_m## and the result depends on the swapping-symmetry of indexes ##k,j##. I can't see if ##j,k## are symmetric or not so I would write out all the components to check.

The square barcket "[" is an anti-symmetrization bracket (Bach bracket)

##V_{[\alpha \beta ]}=\frac{1}{2}\left ( V_{\alpha \beta }-V_{\beta \alpha } \right )##

see
https://physics.stackexchange.com/questions/95133/bracket-notation-on-tensor-indices
 
Last edited:
I think term 4 of upper equation be \Gamma^l_{\ ki}\partial_j v_l. So term 3 + term 4 are symmetric for exchange of j and k. Thus when we subtract the lower from the upper they cancel. Please check it out and let me know please.
 
Last edited:
cozycoz said:
##∇_aT_{bc}=∂_av_{bc}-{Γ^d}_{ac}v_{db}-{Γ^d}_{ab}v_{dc} ##
I assume you are intending that ##T = v## here. If that’s the case, this formula would only hold if ##T_{bc}## is symmetric on ##b## and ##c##. You can see on the RHS the second term has the ##b## in v’s second spot, whereas on the LHS the ##b## is in the first spot. Since in this case, ##T_{bc} = \nabla_b v_c##, ##~## ##T## will in general not be symmetric, so your formula is incorrect. This error was transcribed into your equations, which is what @sweet springs pointed out.
 
  • Like
Likes Orodruin and Mentz114
thanks it helped a lot..
 
OK, so this has bugged me for a while about the equivalence principle and the black hole information paradox. If black holes "evaporate" via Hawking radiation, then they cannot exist forever. So, from my external perspective, watching the person fall in, they slow down, freeze, and redshift to "nothing," but never cross the event horizon. Does the equivalence principle say my perspective is valid? If it does, is it possible that that person really never crossed the event horizon? The...
In this video I can see a person walking around lines of curvature on a sphere with an arrow strapped to his waist. His task is to keep the arrow pointed in the same direction How does he do this ? Does he use a reference point like the stars? (that only move very slowly) If that is how he keeps the arrow pointing in the same direction, is that equivalent to saying that he orients the arrow wrt the 3d space that the sphere is embedded in? So ,although one refers to intrinsic curvature...
So, to calculate a proper time of a worldline in SR using an inertial frame is quite easy. But I struggled a bit using a "rotating frame metric" and now I'm not sure whether I'll do it right. Couls someone point me in the right direction? "What have you tried?" Well, trying to help truly absolute layppl with some variation of a "Circular Twin Paradox" not using an inertial frame of reference for whatevere reason. I thought it would be a bit of a challenge so I made a derivation or...
Back
Top